Những câu hỏi liên quan
Lê Minh Đức
Xem chi tiết
Akai Haruma
9 tháng 10 2017 lúc 17:28

Ta có \(a^2b^2+b^2c^2+c^2a^2\geq a^2b^2c^2\Leftrightarrow \frac{1}{a^2}+\frac{1}{b^2}+\frac{1}{c^2}\geq 1\)

BĐT cần chứng minh tương đương với \(\frac{\frac{1}{c^3}}{\frac{1}{a^2}+\frac{1}{b^2}}+\frac{\frac{1}{b^3}}{\frac{1}{a^2}+\frac{1}{c^2}}+\frac{\frac{1}{a^3}}{\frac{1}{b^2}+\frac{1}{c^2}}\geq \frac{\sqrt{3}}{2}\)

Đặt \((\frac{1}{a},\frac{1}{b},\frac{1}{c})=(x,y,z)\). Bài toán trở thành: 

Cho \(x,y,z>0|x^2+y^2+z^2\geq 1\). CMR \(P=\frac{x^3}{y^2+z^2}+\frac{y^3}{z^2+x^2}+\frac{z^3}{x^2+y^2}\geq \frac{\sqrt{3}}{2}\)

Lời giải:

 Áp dụng BĐT Cauchy -Schwarz:

\(P=\frac{x^4}{xy^2+xz^2}+\frac{y^4}{yz^2+yx^2}+\frac{z^4}{zx^2+zy^2}\geq \frac{(x^2+y^2+^2)^2}{x^2(y+z)+y^2(x+z)+z^2(x+y)}\) (1)

Không mất tính tổng quát, giả sử \(x\geq y\geq z\Rightarrow x^2\geq y^2\geq z^2\) 

Và \(y+z\leq z+x\leq x+y\). Khi đó, áp dụng BĐT Chebyshev: 

\(3[x^2(y+z)+y^2(x+z)+z^2(x+y)]\leq (x^2+y^2+z^2)(y+z+x+z+x+y)\)

\(\Leftrightarrow x^2(y+z)+y^2(x+z)+z^2(x+y)\leq \frac{2(x^2+y^2+z^2)(x+y+z)}{3}\)

Theo hệ quả của BĐT Am-Gm thì: \((x+y+z)^2\leq 3(x^2+y^2+z^2)\Rightarrow x+y+z\leq \sqrt{3(x^2+y^2+z^2)}\)

\(\Rightarrow x^2(y+z)+y^2(x+z)+z^2(x+y)\leq \frac{2(x^2+y^2+z^2)\sqrt{3(x^2+y^2+z^2)}}{3}\) (2)

Từ (1),(2) suy ra \(P\geq \frac{3(x^2+y^2+z^2)^2}{2(x^2+y^2+z^2)\sqrt{3(x^2+y^2+z^2)}}=\frac{\sqrt{3(x^2+y^2+z^2)}}{2}\geq \frac{\sqrt{3}}{2}\)

Ta có đpcm

Dáu bằng xảy ra khi \(x=y=z=\frac{1}{\sqrt{3}}\Leftrightarrow a=b=c=\sqrt{3}\)

Kiệt Nguyễn
5 tháng 5 2020 lúc 12:58

Đặt \(x=\frac{1}{a};y=\frac{1}{b};z=\frac{1}{c}\)

Khi đó giả thiết được viết lại là \(x^2+y^2+z^2\ge1\)và ta cần chứng minh \(\frac{x^3}{y^2+z^2}+\frac{y^3}{z^2+x^2}+\frac{z^3}{x^2+y^2}\ge\frac{\sqrt{3}}{2}\)(*)

Áp dụng BĐT Bunhiacopxki dạng phân thức, ta được:

\(VT_{\left(^∗\right)}=\frac{x^4}{x\left(y^2+z^2\right)}+\frac{y^4}{y\left(z^2+x^2\right)}+\frac{z^4}{z\left(x^2+y^2\right)}\)\(\ge\frac{\left(x^2+y^2+z^2\right)^2}{x\left(y^2+z^2\right)+y\left(z^2+x^2\right)+z\left(x^2+y^2\right)}\)

Đến đây ta đi chứng minh \(\frac{\left(x^2+y^2+z^2\right)^2}{x\left(y^2+z^2\right)+y\left(z^2+x^2\right)+z\left(x^2+y^2\right)}\ge\frac{\sqrt{3}}{2}\)

\(\Leftrightarrow2\left(x^2+y^2+z^2\right)^2\)\(\ge\sqrt{3}\left[x\left(y^2+z^2\right)+y\left(z^2+x^2\right)+z\left(x^2+y^2\right)\right]\)

Ta có: \(x\left(y^2+z^2\right)=\frac{1}{\sqrt{2}}\sqrt{2x^2\left(y^2+z^2\right)\left(y^2+z^2\right)}\)\(\le\frac{1}{\sqrt{2}}\sqrt{\left(\frac{2x^2+y^2+z^2+y^2+z^2}{3}\right)^3}\)

\(=\frac{2\sqrt{3}}{9}\left(x^2+y^2+z^2\right)\sqrt{x^2+y^2+z^2}\)

Tương tự ta có: \(y\left(z^2+x^2\right)\le\frac{2\sqrt{3}}{9}\left(x^2+y^2+z^2\right)\sqrt{x^2+y^2+z^2}\)

\(z\left(x^2+y^2\right)\le\frac{2\sqrt{3}}{9}\left(x^2+y^2+z^2\right)\sqrt{x^2+y^2+z^2}\)

Cộng theo vế của 3 BĐT trên, ta được: 

\(\text{∑}_{cyc}\left[x\left(y^2+z^2\right)\right]\le\frac{2\sqrt{3}}{3}\left(x^2+y^2+z^2\right)\sqrt{x^2+y^2+z^2}\)

\(\Leftrightarrow\sqrt{3}\text{∑}_{cyc}\left[x\left(y^2+z^2\right)\right]\le2\left(x^2+y^2+z^2\right)\sqrt{x^2+y^2+z^2}\)

Cuối cùng ta cần chứng minh được

\(2\left(x^2+y^2+z^2\right)\sqrt{x^2+y^2+z^2}\le2\left(x^2+y^2+z^2\right)^2\)

\(\Leftrightarrow x^2+y^2+z^2\ge1\)(đúng)

Đẳng thức xảy ra khi \(x=y=z=\frac{1}{\sqrt{3}}\Rightarrow a=b=c=\sqrt{3}\)

Khách vãng lai đã xóa
Chàng trai bóng đêm
Xem chi tiết
Pham Quoc Cuong
4 tháng 9 2018 lúc 17:33

\(\frac{2a^2}{a+b^2}=2a-\frac{2ab^2}{a+b^2}\ge2a-\frac{2ab^2}{2b\sqrt{a}}=2a-b\sqrt{a}\ge2a-\frac{b+ba}{2}\) 

Tương tự rồi cộng từng vế ta có: 

\(\frac{2a^2}{a+b^2}+\frac{2b^2}{b+c^2}+\frac{2c^2}{c+a^2}\ge\frac{3}{2}\left(a+b+c\right)-\frac{ab+bc+ca}{2}\) 

Lại có: \(\left(a+b+c\right)^2\left(a^2+b^2+c^2\right)\ge3\left(ab+bc+ca\right)^2\Rightarrow a+b+c\ge ab+bc+ca\) 

\(\Rightarrow VT\ge\frac{3}{2}\left(a+b+c\right)-\frac{a+b+c}{2}\ge a+b+c\) 

Dấu "=' khi a=b=c=1

zZz Cool Kid_new zZz
11 tháng 6 2020 lúc 10:55

Làm 2 cách nhá 

\(\frac{2a^2}{a+b^2}=\frac{2a^2}{\frac{a^2+1}{2}+b^2}=\frac{4a^2}{a^2+2b^2+1}=\frac{4a^4}{a^4+2a^2b^2+a^2}\)

Tương tự rồi theo Cauchy Schwarz ta có được:

\(LHS\ge\frac{\left(2a^2+2b^2+2c^2\right)^2}{a^4+b^4+c^4+2a^2b^2+2b^2c^2+2c^2a^2+3}=\frac{36}{\left(a^2+b^2+c^2\right)^2+3}=\frac{36}{12}=3\)

Đẳng thức xảy ra tại a=b=c=1

Khách vãng lai đã xóa
zZz Cool Kid_new zZz
11 tháng 6 2020 lúc 11:02

Theo BĐT Cauchy Schwarz ta dễ có:

\(LHS=\frac{4a^4}{2a^3+2a^2b^2}+\frac{4b^4}{2b^3+2b^2c^2}+\frac{4c^4}{2c^3+2c^2a^2}\)

\(\ge\frac{\left(2a^2+2b^2+2c^2\right)^2}{2a^3+2b^3+2c^3+2a^2b^2+2b^2c^2+2c^2a^2}\)

\(\ge\frac{36}{\left(a^4+a^2\right)+\left(b^4+b^2\right)+\left(c^4+c^2\right)+2a^2b^2+2b^2c^2+2c^2a^2}\)

\(=\frac{36}{\left(a^2+b^2+c^2\right)^2+a^2+b^2+c^2}\)

\(=3\)

Mà \(3=a^2+b^2+c^2\ge\frac{1}{3}\left(a+b+c\right)^2\Rightarrow a+b+c\le3\Rightarrowđpcm\)

Khách vãng lai đã xóa
Siêu Nhân Lê
Xem chi tiết
Thắng Nguyễn
21 tháng 10 2016 lúc 16:52

Câu hỏi của Tôi Là Ai - Toán lớp 8 - Học toán với OnlineMath

OoO nhóc ngu ngơ OoO dễ...
14 tháng 9 2017 lúc 11:47

Câu hỏi của Tôi Là Ai - Toán lớp 8 - Học toán với OnlineMath

Nguyễn Hữu Ái Linh
14 tháng 9 2017 lúc 11:53

AHAHAHAHAHA!PIKAPIKAPIKA!

thánh yasuo lmht
Xem chi tiết
Thắng Nguyễn
8 tháng 2 2017 lúc 13:26

\(BDT\LeftrightarrowΣ\frac{a^2}{a+b^2}\ge\frac{a+b+c}{2}\)

Áp dụng BDT C-S dạng Engel ta có:

\(Σ\frac{a^2}{a+b^2}=\text{ }Σ\frac{a^4}{a^3+a^2b^2}\ge\frac{\left(a^2+b^2+c^2\right)^2}{Σa^3+a^2b^2}\)

Vậy đi chứng minh \(\frac{\left(a^2+b^2+c^2\right)^2}{Σa^3+a^2b^2}\ge\frac{a+b+c}{2}\)

Hay \(2\left(a^2+b^2+c^2\right)^2\ge\left(a+b+c\right)Σ\left(a^3+b^2c^2\right)\)

\(\hept{\begin{cases}a+b+c=3u\\ab+ac+bc=3v^2\\abc=w^3\end{cases}}\)

Bởi vì điều kiện không phụ thuộc vào \(w^3\), ta thấy rằng bất đẳng thức cuối cùng là một bất đẳng thức tuyến tính của \(w^3\), đủ để chứng minh rằng bất đẳng thức cuối cùng đạt một giá trị cực đại là \(w^3\), xảy ra trong trường hợp hai biến bằng nhau hoặc có thể cho \(w^3\rightarrow0^+\)

Sau khi biến đổi đồng nhất ta cần chứng minh.

 

\(\left(2\left(a^2+b^2+c^2\right)^2-\left(a+b+c\right)\left(a^3+b^3+c^3\right)\right)^2\left(a^2+b^2+c^2\right)\)

\(\ge3\left(a+b+c\right)^2\left(a^2b^2+a^2c^2+b^2c^2\right)^2\)

*)\(b=c=1\) Ta được

\(\left(a-1\right)^2\left(a^8-2a^7+17a^6-8a^5+75a^4-10a^3+73a^2-4a+20\right)\ge0\) ( hiển nhiên đúng)

*)\(w^3\rightarrow0^+\) để  \(c\rightarrow0^+\) và \(b=1\), ta đc:

\(a^{10}-2a^9+10a^8-12a^7+26a^6-26a^5+26a^4-12a^3+10a^2-2a+1\ge0\)( cũng đúng)

Thắng Nguyễn
8 tháng 2 2017 lúc 17:58

cách này phiêu quá lát mk làm lại

Thắng Nguyễn
8 tháng 2 2017 lúc 20:21

Khi \(a+b+c\le\sqrt{3\left(a^2+b^2+c^2\right)}=3\), viết lại BĐT cần chứng minh

\(\frac{a^2}{a+b^2}+\frac{b^2}{b+c^2}+\frac{c^2}{c+a^2}\ge\frac{3}{2}\)

Đặt \(\hept{\begin{cases}a^2=x\\b^2=y\\c^2=z\end{cases}}\) ta có \(x+y+z=3\). Áp dụng AM-GM và Cauchy-Schwarz ta có:

\(Σ\frac{a^2}{a+b^2}=Σ\frac{x}{\sqrt{x}+y}=Σ\frac{x}{\sqrt{\frac{x\left(x+y+z\right)}{3}+y}}\)

\(=Σ\frac{6x}{2\sqrt{3x\left(x+y+z\right)}+6y}\geΣ\frac{6x}{3x+x+y+z+6y}=Σ\frac{6x}{4x+7y+z}\)

\(=Σ\frac{6x^2}{4x^2+7xy+xz}\ge\frac{6\left(x+y+z\right)^2}{Σ\left(4x^2+7xy+xz\right)}=\frac{3}{2}\) 

Hoàn thành!

Đỗ Đức Đạt
Xem chi tiết
l҉o҉n҉g҉ d҉z҉
6 tháng 4 2021 lúc 13:56

Ta có : \(\frac{a}{b^2c^2}+\frac{b}{c^2a^2}+\frac{c}{a^2b^2}=\frac{a^4}{a^3b^2c^2}+\frac{b^4}{b^3c^2a^2}+\frac{c^4}{c^3a^2b^2}\)

Áp dụng bất đẳng thức Cauchy-Schwarz dạng Engel và giả thiết a2 + b2 + c2 = 3abc ta có :

\(\frac{a^4}{a^3b^2c^2}+\frac{b^4}{b^3c^2a^2}+\frac{c^4}{c^3a^2b^2}\ge\frac{\left(a^2+b^2+c^2\right)^2}{a^2b^2c^2\left(a+b+c\right)}=\frac{\left(3abc\right)^2}{a^2b^2c^2\left(a+b+c\right)}=\frac{9}{a+b+c}\left(đpcm\right)\)

Đẳng thức xảy ra <=> a=b=c=1

Khách vãng lai đã xóa
Tiểu Qủy
Xem chi tiết
Trí Tiên亗
20 tháng 8 2020 lúc 15:47

Áp dụng bất đẳng thức Bunhiacopxki dạng phân thức ta được 

\(\frac{a^3}{a+2b}+\frac{b^3}{b+2c}+\frac{c^3}{c+2a}\ge\frac{\left(a^2+b^2+c^2\right)^2}{\left(a+b+c\right)^2}\)

Ta lại có  \(a^2+b^2+c^2\ge\frac{1}{3}\left(a+b+c\right)^2\)

Do đó ta được \(\frac{a^3}{a+2b}+\frac{b^3}{b+2c}+\frac{c^3}{c+2a}\ge\frac{a^2+b^2+c^2}{3}\left(đpcm\right)\)

Đẳng thức xảy ra khi và chỉ khi \(a=b=c\)

p/s: check

Khách vãng lai đã xóa
l҉o҉n҉g҉ d҉z҉
Xem chi tiết
Nguyễn Minh Đăng
7 tháng 5 2021 lúc 19:00

Ta có: 

\(\frac{1}{a^2+2b^2+3}=\frac{1}{\left(a^2+b^2\right)+\left(b^2+1\right)+2}\le\frac{1}{2ab+2b+2}=\frac{1}{2}\cdot\frac{1}{ab+b+1}\)

Tương tự CM được:
\(\frac{1}{b^2+2c^2+3}\le\frac{1}{2}\cdot\frac{1}{bc+c+1}\) và \(\frac{1}{c^2+2a^2+3}\le\frac{1}{2}\cdot\frac{1}{ca+a+1}\)

\(\Rightarrow VT\le\frac{1}{2}\left(\frac{1}{ab+b+1}+\frac{1}{bc+c+1}+\frac{1}{ca+a+1}\right)\)

\(=\frac{1}{2}\left(\frac{1}{ab+b+1}+\frac{ab}{ab^2c+abc+ab}+\frac{b}{abc+ab+b}\right)\)

\(=\frac{1}{2}\left(\frac{1}{ab+b+1}+\frac{ab}{b+1+ab}+\frac{b}{1+ab+b}\right)=\frac{1}{2}\cdot1=\frac{1}{2}\)

Dấu "=" xảy ra khi: a = b = c = 1

Khách vãng lai đã xóa
✦๖ۣۜAugųsť❦❄
7 tháng 5 2021 lúc 19:04

A=\(\frac{1}{a^2+2b^2+3}\)+\(\frac{1}{b^2+2c^2+3}\)+\(\frac{1}{c^2+2a^2+3}\)

ta có: \(\frac{1}{a^2+2b^2+3}\)=\(\frac{1}{\left(a^2+b^2\right)+\left(b^2+1\right)+2}\)\(\le\)\(\frac{1}{2\left(ab+b+1\right)}\)

vì : a2+b2\(\ge\)2\(\sqrt{a^2b^2}\)=2ab

b2+1\(\ge\)2\(\sqrt{b^2x1}\)=2b

cmtt => A\(\le\)\(\frac{1}{2}\)x(\(\frac{1}{ab+b+1}\)+\(\frac{1}{bc+c+1}\)+\(\frac{1}{ca+a+1}\))

=\(\frac{1}{2}\)x(\(\frac{1}{ab+b+1}\)+\(\frac{ab}{ab^2c+abc+ab}\)+\(\frac{b}{cba+ab+b}\))

=\(\frac{1}{2}\)x (\(\frac{1}{ab+b+1}\)+\(\frac{ab}{ab+b+1}\)+\(\frac{b}{ab+b+1}\))=\(\frac{1}{2}\)x\(\frac{ab+b+1}{ab+b+1}\)=\(\frac{1}{2}\)

dấu "=" xảy ra <=> a=b=c=1

Khách vãng lai đã xóa
Lê Minh Vũ
7 tháng 5 2021 lúc 19:05

1a2+2b2+3=1(a2+b2)+(b2+1)+2≤12(ab+b+1)1a2+2b2+3=1(a2+b2)+(b2+1)+2≤12(ab+b+1) . Dấu "=" ⇔a=b=1⇔a=b=1

+ Tương tự : 1b2+2c2+3≤12(bc+c+1)1b2+2c2+3≤12(bc+c+1). Dấu "=" ⇔b=c=1⇔b=c=1

1c2+2a2+3≤12(ca+a+1)1c2+2a2+3≤12(ca+a+1). Dấu "=" c=a=1c=a=1

Do đó : VT≤12(1ab+b+1+1bc+c+1+1ca+a+1)=12(1ab+b+1+ababc⋅b+abc+ab+babc+ab+b)VT≤12(1ab+b+1+1bc+c+1+1ca+a+1)=12(1ab+b+1+ababc⋅b+abc+ab+babc+ab+b)

=12(1ab+b+1+abab+b+1+bab+b+1)=12=12(1ab+b+1+abab+b+1+bab+b+1)=12

Dấu "=" ⇔a=b=c=1

Khách vãng lai đã xóa
Tường Nguyễn Thế
Xem chi tiết
Nguyễn Việt Lâm
18 tháng 11 2019 lúc 15:02

\(VT=\frac{b^2c^2}{b+c}+\frac{a^2c^2}{a+c}+\frac{a^2b^2}{a+b}\ge\frac{\left(ab+bc+ca\right)^2}{2\left(a+b+c\right)}\ge\frac{3abc\left(a+b+c\right)}{2\left(a+b+c\right)}=\frac{3}{2}\)

Dấu "=" xảy ra khi \(a=b=c=1\)

Khách vãng lai đã xóa
Trần Lâm Thiên Hương
Xem chi tiết
Trần Lâm Thiên Hương
15 tháng 5 2018 lúc 21:03

Mình nhầm, phải là \(\le\frac{1}{3}\)mọi người làm giúp mình với mình cần gấp

zZz Cool Kid_new zZz
1 tháng 8 2020 lúc 19:31

Theo BĐT Cauchy Schwarz và các biến đổi cơ bản ta dễ có được:
\(\frac{a^2}{\left(2a+b\right)\left(2a+c\right)}=\frac{a^2}{2a\left(a+b+c\right)+2a^2+bc}=\frac{1}{9}\left[\frac{\left(2a+a\right)^2}{2a\left(a+b+c\right)+2a^2+bc}\right]\)

\(\le\frac{1}{9}\left[\frac{4a^2}{2a\left(a+b+c\right)}+\frac{a^2}{2a^2+bc}\right]=\frac{1}{9}\left(\frac{2a}{a+b+c}+\frac{a^2}{2a^2+bc}\right)\)

\(\Rightarrow LHS\le\frac{1}{9}\left(2+\frac{a^2}{2a^2+bc}+\frac{b^2}{2b^2+ca}+\frac{c^2}{2c^2+ab}\right)\)

Tiếp tục theo BĐT Cauchy Schwarz dạng Engel:

\(\frac{a^2}{a^2+2bc}+\frac{b^2}{b^2+2ca}+\frac{c^2}{c^2+2ab}\ge\frac{\left(a+b+c\right)^2}{\left(a+b+c\right)^2}=1\)

Ta thực hiện phép đổi biến thì:

\(\frac{ab}{ab+2c^2}+\frac{bc}{bc+2a^2}+\frac{ca}{ca+2b^2}\ge1\)

Đến đây là phần của bạn

Khách vãng lai đã xóa
tth_new
3 tháng 8 2020 lúc 19:10

(Vào thống kê hỏi đáp xem ảnh nhé! 2 cách, cách đầu dùng kỹ thuật uvw, cách kia là SOS)

Khách vãng lai đã xóa